6
$\begingroup$

Let $M$ be some large real number and $\delta>0$. I would like to estimate the number of solutions for the inequality

$$|\sqrt{n_1}+\sqrt{n_2}-\sqrt{n_3}-\sqrt{n_4}|<\delta\sqrt{M},$$

where $n_i$ are positive integers with $M<n_i\leq 2M$ for all $1\leq i\leq 4$. Let us denote this quantity by $N(M,\delta)$.

There is a paper by O.Robert and P.Sargos which provides the bound for a bit more general quantity. In our case their work gives

$$N(M,\delta)\ll M^{2+o(1)}+\delta M^{4+o(1)}.$$

I think, some heuristics suggest that when $\delta\gg 1$ this bound is more or less optimal. But I'm interested in the case when $\delta=o\left(\frac{1}{M}\right)$. So, my question is: can the bound of Robert and Sargos be improved for small $\delta$ and when should we expect that $N(M,\delta)\ll M^{2+o(1)}$? For example, we always have $N(M,M^{-2})\ll M^{2+o(1)}$ but can we do better (i.e. prove this bound for larger $\delta$) or at least are there any conjectures about this?

$\endgroup$

1 Answer 1

5
$\begingroup$

For $\delta\geq 1$, the inequality is automatic, hence $N(M,\delta)=M^4$.

For $\delta<1$, the bound by Robert and Sargos is optimal up to the $o(1)$ in the exponent, and actually they mention this right before their Theorem 1. They do not give the proof, but the claim follows easily from their Lemma 1, which is a special case of Lemma 2.1 in Watt: Exponential sums and the Riemann zeta-function II (J. London Math. Soc. 39 (1989), 385-404.). For convenience, I give a proof from scratch, using Watt's ideas.

We assume $\delta<1$. It is clear that $$N(M,\delta)\geq\sum_{M<m_1,m_2,m_3,m_4\leq 2M}\max\left(0,1-\left|\frac{\sqrt{m_1}+\sqrt{m_2}-\sqrt{m_3}-\sqrt{m_4}}{\delta\sqrt{M}}\right|\right).$$ By Fourier analysis (cf. triangular function), the $\max(0,\dots)$ term on the right hand side equals $$\int_{-\infty}^\infty\left(\frac{\sin\pi t}{\pi t}\right)^2\ e\left(\frac{\sqrt{m_1}+\sqrt{m_2}-\sqrt{m_3}-\sqrt{m_4}}{\delta\sqrt{M}}\,t\right)dt,$$ where $e(x):=\exp(2\pi ix)$ as customary in analytic number theory. Hence, introducing the exponential generating function $$ f(x):=\sum_{M<m\leq 2M}e\bigl(\sqrt{m}x\bigr),$$ we can rewrite the above lower bound as $$N(M,\delta)\geq\int_{-\infty}^\infty\left(\frac{\sin\pi t}{\pi t}\right)^2\ \left|f\left(\frac{t}{\delta\sqrt{M}}\right)\right|^4\,dt.$$ The integrand is nonnegative, and for $|t|<\delta/\sqrt{128}$ it exceeds $6M^4/25$. The latter is because, for $|x|<1/\sqrt{128M}$, each term in $f(x)$ has real part exceeding $1/\sqrt{2}$, so that $|f(x)|>M/\sqrt{2}$. We conclude that $$N(M,\delta)>\frac{2\delta}{\sqrt{128}}\cdot\frac{6M^4}{25}>\frac{\delta M^4}{24}.$$ On the other hand, it is obvious that $N(M,\delta)\geq M^2$, hence we proved for $\delta<1$ that $$N(M,\delta)>\frac{M^2+\delta M^4}{25}.$$

$\endgroup$

Your Answer

By clicking “Post Your Answer”, you agree to our terms of service and acknowledge you have read our privacy policy.

Not the answer you're looking for? Browse other questions tagged or ask your own question.